You invest AED 5000 in a saving account that earns 4% interest each year. If you do not use the account for a year, how much money will you have in your account after a year?

Answers

Answer 1

Answer:

5200 AED or 1404 USD

Step-by-step explanation:

You have 5000 in your account. If your money grows 4% each year, it's basically like multiplying by 1.04.

5000 * 1.04 = 5200

If the question is asking in USD, the conversion rate from AED to USD is

AED * 0.27 = USD

We can substitute

5200 AED* 0.27 = 1404 USD.


Related Questions

plz help very easy i pormise

Answers

Answer:

8/90

Step-by-step explanation:

hope this helps

Answer:

It's 38/45

Step-by-step explanation:

I don't really have a step-by-step explanation. I just looked it up! ;) good luck

1. Andrea’s car has broken down for the third time this month, and she has decided to buy a different car. Her dream car, a 2016 Chevrolet Camaro, costs $38,000. She has a $4,000 down payment saved, and the bank is offering 3.00% APR on 60 month loans for new cars.

a. How much will Andrea need to borrow to buy the Camaro?

b. What would Andrea need to type in Excel to figure her monthly payment, and what will her monthly payment be?

c. What will the total of all of Andrea’s payments be over the lifetime of the car loan?

d. How much will Andrea pay in interest over the lifetime of the car loan?

Answers

A) Andrea will need to borrow $34,000 to buy the Camaro

B) Andrea will need to type =(34000x1.03 ^ 5) / 60 in Excel to figure her monthly payment, which will be $656,92.

C) The total of all of Andrea’s payments be over the lifetime of the car loan will be $39.415,31

D) Andrea will pay $5415.31 in interest over the lifetime of the car loan

Since Andrea's car has broken down for the third time this month, and she has decided to buy a different car, and her dream car, a 2016 Chevrolet Camaro, costs $ 38,000, and she has a $ 4,000 down payment saved, and the bank is offering 3.00% APR on 60 month loans for new cars, to determine A) how much will Andrea need to borrow to buy the Camaro; B) what would Andrea need to type in Excel to figure her monthly payment from her, and what will her her monthly payment be from her; C) what will the total of all of Andrea’s payments be over the lifetime of the car loan; and D) how much will Andrea pay in interest over the lifetime of the car loan, the following calculations must be performed:

A) 38000 - 4000 = X 34000 = X

B) = (34000x1.03 ^ 5) / 6039.415,31 / 60656,92

C) 34000 x 1.03^5 = X34000 x 1.1592 = X39.415,31 = X

D) (34000 x 1.03^5) - 34000 = X(34000 x 1.1592) - 34000 = X39415.31 - 34000 = X5415.31 = X

Learn more in https://brainly.com/question/14295570

PLEASE HELP!!
Guys please help i need this

Answers

Answer:

Because they are both base angles of isosceles triangles

Which graph represents 6x-2y>-11

Answers

Answer:

Answer:

C

Step-by-step explanation:

Remember when using the inequality sign < or > the line will be dotted so we can immediately eliminate answer choices b and d

Now let us solve for y

6x-2y>-11

step 1 subtract each side by 6x

now we have

-2y>-11-6x

step 2 divide each side by -2

-11-/2=11/2

-6/-2=3

IMPORTANT -when you divide inequalities by a negative number you flip the inequality sign

So now we have

See image below for reference

When the inequality is facing x the solutions are going to be on top of the line

hope this helps!

Step-by-step explanation:

There are integers that are not rational
numbers, true or false?

Answers

Answer:

integers are always rational. They don't include decimals

Step-by-step explanation:

Answer:

False

Step-by-step explanation:

All integers are rational numbers, since they can be expressed as a ratio (fraction) with a denominator of 1.

Use set notation to write the members of the following set, or state that the set has no members. Odd numbers between 2 and 82 that are multiples of 9

Answers

Let the set of all Odd multiples of 9 between 2 and 82 be denoted by D, then, using set-builder notation,

[tex]D=\{ 18n+9 \mid n \in \mathbb{N}, 0\le n\le 4 \}[/tex]

The odd multiples of 9, [tex]m[/tex], in the range [tex]2\le m \le 82[/tex] form the set

[tex]\{9,27,45,63,81\}[/tex]

Each member of the set is a term of the arithmetic progression

[tex]U_n=18n+9[/tex]

where the values of [tex]n[/tex] range from 0 to 4, or [tex]0\le n\le 4[/tex]

Putting these facts together, we get the result

[tex]D=\{ 18n+9 \mid n \in \mathbb{N}, 0\le n\le 4 \}[/tex]

Learn more about set-builder notation here: https://brainly.com/question/17238769

In what time will $1000 amount to $1331 at 10% per annum, compounded annually?​

Answers

The time is 3 years


Step by step explanation:
Answer:

3 Years

Step-by-step explanation:

Compound interest is a multiplier that affects the new value each time.

If the compound interest is 10% per annum, the multiplier each year is 1.1.

Year 1: $1000 * 1.1 = $1100

Year 2: $1100* 1.1 = $1210

Year 3: $1210* 1.1 = $1331

This means that it will take 3 years for $1000 to amount to $1331 at 10% per annum.

Alternatively, you can use a logarithm. The calculation can be written as:

1000 * [tex]1.1^n[/tex] = 1331, where n is the number of years.

Rearranged, this gives [tex]1.1^n[/tex] = 1331/1000 or [tex]1.1^n[/tex] = 1.331.

A logarithm can work out the value of n, and is written in the form log1.1(1.331), which gives a value of 3.

rewrite the expression 44 + 32 + 112 using the distributive property​

Answers

Answer:

4(11+8+28)

Step-by-step explanation:

44 + 32 + 112

4(11+8+28)

I found the GCF of all three numbers.

Hope this helps :)

consider the graph of a linear equation. see below , which statement is true?

Answers

Answer:

B

Step-by-step explanation:

If you go to -4, 0 the line intersects that point

-36.12 divided by -5 3/5

Answers

[tex] - 36.12 \div - 5 \frac{3}{5} \\ - 36.12 \div - \frac{28}{5} \\ = 6.45[/tex]

- BRAINLIEST answerer

Please help! (Urgent)
Only answer if correct.

I will be asking more so stay tuned.

Answers

Answer:

Number of rotations: 5

Slope: 1/2

Step-by-step explanation:

Answer: why did you ask twice for the same problem

Step-by-step explanation:

Ryan drove 205 miles in 5 hours. If he drove at a constant rate, how far did he travel in one hour?

Answers

Answer: 41 miles

Step-by-step explanation:

Answer:

41 miles

Step-by-step explanation:

41 miles in a hour rate.

HELP ASAP PLSSSSS I NEED IT

Answers

Answer: B) 5/24

========================================================

Explanation:

The notation [tex]P(\text{blue, not 2})[/tex] is asking for the probability of landing on a blue space and having the dice show a value that isn't 2. That means we can roll any of the following: {1,3,4,5,6}.

The probability of landing on blue is 1/4 since there's one blue space out of four total. This is assuming all four spaces are equal in area. The probability of landing on anything but "2" is 5/6 because there are five sides that aren't labeled "2" out of six sides total.

Multiply the fractions found: (1/4)*(5/6) = (1*5)/(4*6) = 5/24

Needing to find the answer to this question attached. Thank you

Answers

[tex]\sin \theta -1 =0\\\\\ \implies \sin \theta = 1\\\\ \implies \theta = \dfrac{\pi}2 + 2\pi n\\\\\text{For n =0,}\\\\\theta = \dfrac{\pi}2[/tex]

Elizabeth recently bought a new bike, and she has started biking to and from work. The
round-trip distance is 9 kilometers.
Write an equation that shows the relationship between the number of round trips, x, and
the total number of kilometers Elizabeth has biked, y.

Answers

Answer:

the answer is  9x

Step-by-step explanation:

y=9x is the relationship between the number of round trips, x, and the total number of kilometers Elizabeth has biked, y.

What is Equation?

Two or more expressions with an Equal sign is called as Equation.

Given that,

Elizabeth recently bought a new bike, and she has started biking to and from work.

The round-trip distance is 9 kilometers.

We need to find an equation that shows the relationship between the number of round trips, x, and the total number of kilometers Elizabeth has biked, y.

y=9x

Hence y=9x is the relationship between the number of round trips, x, and the total number of kilometers Elizabeth has biked, y.

To learn more on Equation:

https://brainly.com/question/10413253

#SPJ2

$32.00for a 14 2/9 km taxi ride. What is the cost per kilometer

Answers

[tex]\begin{array}{ccll} \$&km\\ \cline{1-2} 32&14\frac{2}{9}\\[1em] x&1 \end{array}\implies \cfrac{32}{x}=\cfrac{14\frac{2}{9}}{1}\implies \cfrac{32}{x}=14\frac{2}{9}\implies \cfrac{32}{x}=\cfrac{14\cdot 9+2}{9} \\\\\\ \cfrac{32}{x}=\cfrac{128}{9}\implies 288=128x\implies \cfrac{288}{128}=x\implies \stackrel{~\hfill \textit{2 bucks and 25 cents}}{\cfrac{9}{4}=x\implies 2\frac{1}{4}}=x[/tex]

solve using quadratic formula
6x^2-x=2

Answers

[tex]6x^2-x=2\implies 6x^2-1x-2=0 \\\\[-0.35em] ~\dotfill\\\\ ~~~~~~~~~~~~\textit{quadratic formula} \\\\ \stackrel{\stackrel{a}{\downarrow }}{6}x^2\stackrel{\stackrel{b}{\downarrow }}{-1}x\stackrel{\stackrel{c}{\downarrow }}{-2}=0 \qquad \qquad x= \cfrac{ - b \pm \sqrt { b^2 -4 a c}}{2 a}[/tex]

[tex]x = \cfrac{ -(-1) \pm \sqrt { (-1)^2 -4(6)(-2)}}{2(6)}\implies x = \cfrac{1\pm\sqrt{1+48}}{12} \\\\\\ x = \cfrac{1\pm\sqrt{49}}{12}\implies x = \cfrac{1\pm 7}{12}\implies x = \begin{cases} \frac{8}{12}\to &\frac{2}{3}\\[1em] -\frac{6}{12}\to &-\frac{1}{2} \end{cases}[/tex]

will give 25 points if right

Answers

The slope of the equation is 1/6

Answer:

1/6

Step-by-step explanation:

When the equation is in the form "y = mx + b", m is the slope. This linear line has a slope of 1/6 and a y-intercept of -1/2.

How would I write 129.701 in expanded form

Answers

100+20+9+0.7+0.1 i think goodluck

What’s the slope of 10,6 and 4, 1.2

Answers

let's firstly change the 1.2 to a fraction

[tex]1.\underline{2}\implies \cfrac{12}{1\underline{0}}\implies \cfrac{6}{5} \\\\[-0.35em] ~\dotfill\\\\ (\stackrel{x_1}{10}~,~\stackrel{y_1}{6})\qquad (\stackrel{x_2}{4}~,~\stackrel{y_2}{\frac{6}{5}}) \\\\\\ \stackrel{slope}{m}\implies \cfrac{\stackrel{rise} {\stackrel{y_2}{\frac{6}{5}}-\stackrel{y1}{6}}}{\underset{run} {\underset{x_2}{4}-\underset{x_1}{10}}}\implies \cfrac{~~ \frac{6-30}{5}~~}{-6}\implies \cfrac{~~ \frac{-24}{5}~~}{-6}\implies \cfrac{~~ -\frac{24}{5}~~}{-\frac{6}{1}}[/tex]

[tex]-\cfrac{\stackrel{4}{~~\begin{matrix} 24 \\[-0.7em]\cline{1-1}\\[-5pt]\end{matrix}~~}}{5}\cdot -\cfrac{1}{\underset{1}{~~\begin{matrix} 6 \\[-0.7em]\cline{1-1}\\[-5pt]\end{matrix}~~}}\implies \boxed{\cfrac{4}{5}}[/tex]

Solve
-4(-2)

A. -8

B. -6

C. -2

D. 8

Answers

Answer:

D

..............................

A restaurant has an electronic system that randomly selects customers when they pay for their meal to receive a coupon for their next visit. Each customer has a 10\%10%10, percent probability of being selected to receive a coupon, and one customer being selected or not doesn't affect whether or not another customer will be selected.
Suppose that 222 customers pay for their meals, one after the other.
What is the probability that NEITHER customer is selected to receive a coupon?
Round your answer to two decimal places.

Answers

Answer:

Step-by-step explanation:

Using the binomial distribution, it is found that there is a 0.81 = 81% probability that NEITHER customer is selected to receive a coupon.

For each customer, there are only two possible outcomes, either they receive the coupon, or they do not. The probability of a customer receiving the coupon is independent of any other customer, which means that the binomial distribution is used to solve this question.

Binomial probability distribution

0.81 = 81% probability that NEITHER customer is selected to receive a coupon.

Which of the following is the greatest common factor of 48 and 32?

Answers

Answer:

8

Step-by-step explanation:

cus...

On October 1, Organic Farming purchases wind turbines for $140,000. The wind turbines are expected to last six years, have a salvage
value of $20,000, and be depreciated using the straight-line method.
1. Compute depreciation expense for the last three months of the first year.
2. Compute depreciation expense for the second year.

Answers

9514 1404 393

Answer:

$5000$20,000

Step-by-step explanation:

1.

The annual depreciation is the predicted change in value each year. The change in value for 6 years is predicted to be ...

  $140,000 -20,000 = $120,000 . . . . loss of value in 6 years

3 months is 1/4 of one year, so the loss of value in that time period is ...

  $120,000/6 × 1/4 = $5000

The depreciation expense for the last 3 months of the first year is $5000.

__

2.

The depreciation expense for the second year is ...

  $120,000/6 = $20,000

which graph be represents the soulution set of y[tex]\leq[/tex]3/4x-4?

Answers

the graph would intercept at (0,-4) on the y-axis & at (5.3,0) on the x-axis. And the shaded region would be on the right side of the line on the graph.

Should the United States return to a gold standard? Why?

Answers

Answer:

No

Step-by-step explanation:

If the U.S goes in a gold standard it can put the whole state in a crises

2-2 this is a freebie for points guys

Answers

Answer:

2-2=0

Step-by-step explanation:

You have 2 cookies and you give it to 2 friends.

Answer:

0

Step-by-step explanation:

According to the Laws of Physics, yeah I don't wanna try explaining this

The school football team is hosting a fundraiser dinner. Tickets are $20 for adults, $15 for seniors, and $6 for children. The expression 20a + 15s + 6c represents the total sales for a adult, s senior tickets, and c child tickets. Mariel sold 6 adult, 4 senior, and 8 child tickets.

20a + 15s + 6c = 20 (6) + 15 (4) + 6 (8) = ? + ? + ?

Answers

20(6) + 15(4) + 6(8)
120 + 60 + 48
228

What is the better buy?
16 oz of goldfish for $4.75
9 oz of goldfish for $2.50

Answers

Answer:

Maybe 9 oz, if I’m wrong sorry! D:

Suppose y = sqrt 2x+1, where x and y are functions of t.
(a) If dx/dt = 9, find dy/dt when x = 4.
(b) If dy/dt = 3, find dx/dt when x = 40.

Answers

If y = √(2x + 1), then differentiating both sides implicitly with respect to t gives

dy/dt = 1/2 • 1/√(2x + 1) • 2 • dx/dt = 1/√(2x + 1) • dx/dt

(a) If dx/dt = 9 and x = 4, then

dy/dt = 1/√(2•4 + 1) • 9

dy/dt = 1/√(8 + 1) • 9

dy/dt = 1/√9 • 9

dy/dt = 9/3

dy/dt = 3

(b) If dy/dt = 3 and x = 40, then

3 = 1/√(2•40 + 1) • dx/dt

3 = 1/√(80 + 1) • dx/dt

3 = 1/√81 • dx/dt

3 = 1/9 • dx/dt

dx/dt = 27

Other Questions
If the radius of a circle equals 20 cm find its circumference correct to one decimal place What is friction ??? what do we call judgments about people, situations, objects, or thoughts? solve for q q/-2+17=13 q= A jet is flying in a direction N 70 E with a speed of 600 mi/h. Find the north and east components of the velocity. Round to two decimal places. What is a possible pH of acid rain?07.06.904.310.5 Whats the product of 2.6 do you believe that drug defense / abuse in the philippines is decreasing explain your answer A cell uses active transport to move sodium ions out of the cell. Why does the cell use active transport instead of diffusion to move sodium ions out of the cell?. How to turn robot voice like real human voice. What is the weight of a 68-kg astronaut in outer space traveling with constant velocity?. Kima is an athlete and weighs more than her sister, who is of the same height. What could be the reason?OA.Kima has more muscle mass than her sister because she is more athletic..As part of her athletic regimen, Kima takes several supplements that add to her weight.O c.To fuel her high level of activity, Kima consumes extra calories and carbohydrates. 2.) What native chief opposed American expansion and aimed to stop it?a.) Blue Jacketb.) Tecumsehc.) The Prophetd.) Chief Joseph Youssef and Sophia are driving in two different cars and not paying attention to their surroundings. Youssef approaches an intersection moving 11 m/s East while Sophia approaches the same intersection moving 13.5 m/s North. Both speeds are measured relative to the ground. When Youssef is 220 m from the intersection, Sophia is 275 m from the intersection. Do the two cars collide when they reach the intersection what are the four functions that are increasing on the interval If Brianna spent $65 on necklaces and bracelets at a crafts fair.She spent $23 on bracelets and the rest on necklaces.If the necklaces all cost the same and Brianna bought 3, how much did each necklace cost? When resting, a spring measures 73.3 millimeters. When the spring iscompressed, it measures 51.29 millimeters. Find the percent ofdecrease in the length of the spring. Round to the nearest percent ifnecessary The table gives information about the heights of 50 trees. Work out an estimate for the mean height of the trees. What is the problem for not sleeping? The lithosphere is made up of many separate plate called